1. Trang chủ
  2. » Tài Chính - Ngân Hàng

TEST BANK management advisory services by tan p03 working capital finance

58 2.2K 1

Đang tải... (xem toàn văn)

Tài liệu hạn chế xem trước, để xem đầy đủ mời bạn chọn Tải xuống

THÔNG TIN TÀI LIỆU

MANAGEMENT ADVISORY SERVICES WORKING CAPITAL FINANCE WORKING CAPITAL MANAGEMENT Net Working Capital During 1990, Mason Company’s current assets increased by $120, current liabilities decreased by $50, and net working capital (E) a Increased by $70 c Decreased by $170 b Did not change d Increased by $170 CMA 1290 1-19 During the year, Mason Company's current assets increased by $130, current liabilities decreased by $60, and net working capital (E) A Increased by $70 C Decreased by $190 B Did not change D Increased by $190 Gleim A service enterprise's working capital at the beginning of January was $70,000 The following transactions occurred during January: Performed services on account $30,000 Purchased supplies on account 5,000 Consumed supplies 4,000 Purchased office equipment for cash 2,000 Paid short-term bank loan 6,500 Paid salaries 10,000 Accrued salaries 3,500 What is the amount of working capital at the end of January? (M) A $80,500 C $50,500 B $78,500 D $47,500 CIA 1193 IV-36 The following are the January and June 30 balance sheets of a company: Assets (in millions) Jan Cash $3 Accounts receivable Inventories Fixed assets 10 Total assets $26 Accounts payable $2 Notes payable Accrued wages Long-term debt Stockholder's equity 10 Total liabilities and stockholder's equity $26 CMA EXAMINATION QUESTIONS June 30 $4 10 11 $29 $3 11 10 $29 From January to June 30, the net working capital (M) A Decreased by $1 million C Increased by $1 million B Stayed the same D Increased by $2 million CIA 1192 IV-52 Covenant Limitation Maximum Loan Availment * It is the policy of Franz Corp that the current ratio cannot fall below 1.5 to 1.0 Its current liabilities are P400,000 and the present current ratio is to How much is the maximum level of new short-term loans it can secure without violating the policy? (M) a P400,000 c P266,667 b P300,000 d P800,000 RPCPA 1096 A firm's current ratio is currently 1.70 to Management knows it cannot violate a working capital restriction contained in its bond indenture If the firm's current ratio falls below 1.40 to 1, technically it will have defaulted If current liabilities are $200 million, the maximum new commercial paper that can be issued to finance inventory expansion is (M) A $80 million C $150 million B $370 million D $280 million Gleim A firm's current ratio is currently 1.75 to Management knows it cannot violate a working capital restriction contained in its bond indenture If the firm's current ratio falls below 1.5 to 1, technically it will have defaulted If current liabilities are $250 million, the maximum new commercial paper that can be issued to finance inventory expansion is (M) A $375.00 million C $562.50 million B $125.00 million D $437.50 million CMA 0683 1-8, 1292 1-23 Management of a company does not want to violate a working capital restriction contained in its bond indenture If the firm's current ratio falls below 2.0 to 1, technically it will have defaulted The firm's current ratio is now 2.2 to If current liabilities are $200 million, the maximum new commercial paper that can be issued to finance inventory expansion is (M) A $20 million C $240 million B $40 million D $180 million Gleim Iken Berry Farms has $5 million in current assets, $3 million in current liabilities, and its initial inventory level is $1 million The company plans to increase its inventory, and it will raise additional short-term debt (that will show up as notes payable on the balance sheet) to purchase the inventory Assume that the value of the remaining current assets will not change The company’s bond covenants require it to maintain a current ratio that is greater than or equal to Page of 58 MANAGEMENT ADVISORY SERVICES WORKING CAPITAL FINANCE 1.5 What is the maximum amount that the company can increase its inventory before it is restricted by these covenants? a $0.50 million d $1.66 million b $1.00 million e $2.33 million c $1.33 million Brigham Maximum Cash Dividend MFC Corporation has 100,000 shares of stock outstanding Below is part of MFC’s Statement of Financial Position for the last fiscal year MFC Corporation Statement of Financial Position – Selected Items December 31, 1996 Cash Accounts receivable Inventory Prepaid assets $455,000 900,000 650,000 45,000 Accrued liabilities 285,000 Accounts payable 550,000 Current portion, long-term notes payable 65,000 What is the maximum amount MFC can pay in cash dividends per share and maintain a minimum current ratio of to 1? Assume that all accounts other than cash remain unchanged (M) a $2.05 c $3.35 b $2.50 d $3.80 CMA 0697 1-16 Effect of Plant Expansion on Working Capital 10 Shaw Corporation is considering a plant expansion that will increase its sales and net income The following data represent management’s estimate of the impact the proposal will have on the company: Current Proposal Cash $ 100,000 $ 120,000 Accounts payable 350,000 430,000 Accounts receivable 400,000 500,000 Inventory 380,000 460,000 Marketable securities 200,000 200,000 Mortgage payable (current) 175,000 325,000 CMA EXAMINATION QUESTIONS Fixed assets 2,500,000 Net income 500,000 The effect of the plant expansion of Shaw’s working capital will be a(n) (M) a Decrease of $150,000 c Increase of $30,000 b Decrease of $30,000 d Increase of $120,000 3,500,000 650,000 CMA 1292 1-22 11 Finan Corporation's management is considering a plant expansion that will increase its sales and have commensurate impact on its net working capital position The following information presents management's estimate of the impact the proposal will have on Finan Current Proposal Cash $ 100,000 $ 110,000 Accounts payable 400,000 470,000 Accounts receivable 560,000 690,000 Inventory 350,000 380,000 Marketable securities 200,000 200,000 Fixed assets 2,500,000 3,500,000 Net income 500,000 650,000 The impact of the plant expansion on Finan's working capital would be (M) A A decrease of $100,000 C An increase of $100,000 B A decrease of $950,000 D An increase of $950,000 CMA 1286 1-29 12 The Herb Salter Corporation is considering a plant expansion that will increase its sales and net income The following data represent management's estimate of the impact the proposal will have on the company: Current Proposed Cash $ 120,000 $ 140,000 Accounts payable 360,000 450,000 Accounts receivable 400,000 550,000 Inventory 360,000 420,000 Marketable securities 180,000 180,000 Mortgage payable (current) 160,000 310,000 Fixed assets 2,300,000 3,200,000 Net income 400,000 550,000 The effect of the plant expansion on Salter's working capital will be a(n) (M) A Increase of $240,000 C Increase of $230,000 B Decrease of $10,000 D Increase of $10,000 Gleim Page of 58 MANAGEMENT ADVISORY SERVICES WORKING CAPITAL FINANCING POLICY Moderate 13 Wildthing Amusement Company’s total assets fluctuate between $320,000 and $410,000, while its fixed assets remain constant at $260,000 If the firm follows a maturity matching or moderate working capital financing policy, what is the likely level of its long-term financing? (E) a $ 90,000 d $410,000 b $260,000 e $320,000 c $350,000 Brigham Conservative 23 Great Company has P8,000,000 in current assets, P3,500,000 of which are considered permanent current assets In addition, the firm has P6,000,000 invested in fixed assets Great Company wishes to finance all fixed assets and permanent current assets plus half of its temporary current assets with long-term financing costing 15% Short-term financing currently costs 10% Great Company’s earnings before interest and taxes are P2,200,000 Income tax rate is 40% How much would Real Company’s earnings after taxes be under this financing plan? A P112,500 C P225,000 B P127,500 D P85,000 Pol Bobadilla Aggressive 49 Normal Company has total fixed assets of P100,000 and no current liabilities The table below displays its wide variation in current asset components: 1st Qtr 2nd Qtr 3rd Qtr 4th Qtr Cash P 20,000 P 10,000 P 15,000 P 20,000 Accounts receivable 66,000 25,000 47,000 88,000 Inventory 20,000 65,000 59,000 10,000 Total P106,000 P100,000 P121,000 P118,000 If Normal’s policy is to finance all fixed assets and half the permanent current assets with longterm financing and the rest with short-term financing, what is the level of long-term financing? (D) A P68,000 C P150,000 B P100,000 D P155,625 Pol Bobadilla Working Capital Policy Options 14 Mason Company's board of directors has determined options to increase working capital next year Option is to increase current assets by $120 and decrease current liabilities by $50 Option is to increase current assets by $180 and increase current liabilities by $30 CMA EXAMINATION QUESTIONS WORKING CAPITAL FINANCE Option is to decrease current assets by $140 and increase current liabilities by $20 Option is to decrease current assets by $100 and decrease current liabilities by $75 Which option should Mason choose to maximize net working capital? A Option C Option B Option D Option Gleim 15 Jarrett Enterprises is considering whether to pursue a restricted or relaxed current asset investment policy The firm’s annual sales are $400,000; its fixed assets are $100,000; debt and equity are each 50 percent of total assets EBIT is $36,000, the interest rate on the firm’s debt is 10 percent, and the firm’s tax rate is 40 percent With a restricted policy, current assets will be 15 percent of sales Under a relaxed policy, current assets will be 25 percent of sales What is the difference in the projected ROEs between the restricted and relaxed policies? (M) a 0.0% d 1.6% b 6.2% e 3.8% c 5.4% Brigham Comprehensive Questions 56 thru 61 are based on the following information Gitman Irish Air Services has determined several factors relative to its asset and financing mix  The firm earns 10 percent annually on its current assets  The firm earns 20 percent annually on its fixed assets  The firm pays 13 percent annually on current liabilities  The firm pays 17 percent annually on long-term funds  The firm's monthly current, fixed and total asset requirements for the previous year are summarized in the table below: Month Current Assets Fixed Assets Total Assets January $45,000 $100,000 $145,000 February 40,000 100,000 140,000 March 50,000 100,000 150,000 April 55,000 100,000 155,000 May 60,000 100,000 160,000 June 75,000 100,000 175,000 July 75,000 100,000 175,000 August 75,000 100,000 175,000 September 60,000 100,000 160,000 October 55,000 100,000 155,000 November 50,000 100,000 150,000 December 50,000 100,000 150,000 Page of 58 MANAGEMENT ADVISORY SERVICES WORKING CAPITAL FINANCE B 3:1 56 The firm's monthly average permanent funds requirement is (E) A $100,000 C $140,000 B $57,500 D $157,500 63 The firm's initial net working capital is A -$ 5,000 B $13,000 57 The firm's monthly average seasonal funds requirement is (M) A $17,500 C $40,000 B $57,500 D $157,500 65 If the firm was to shift $3,000 of current assets to fixed assets, the firm's net working capital would _, the annual profits on total assets would _, and the risk of technical insolvency would _, respectively A increase; decrease; increase C increase; decrease; decrease B decrease; increase; decrease D decrease; increase; increase 59 The firm's annual financing costs of conservative financing strategy is (M) A $22,775 C $29,750 B $26,075 D $21,175 60 The firm's annual profits on total assets for the previous year was (M) A $20,000 C $23,625 B $21,500 D $25,750 61 If the firm's current liabilities in December were $40,000, the net working capital was (E) A $140,000 C $10,000 B $60,000 D -$10,000 Assets Current assets Fixed assets Gitman Liabilities & Equity Current Liabilities $ 5,000 Long-term debt 12,000 Equity 13,000 Total $30,000 Total $30,000 The company earns percent on current assets and 15 percent on fixed assets The firm's current liabilities cost percent to maintain and the average annual cost of long-term funds is 20 percent $10,000 20,000 62 The firm's initial ratio of current to total asset is _ A 1:3 C 2:3 CMA EXAMINATION QUESTIONS C $ 5,000 D $10,000 64 The firm's initial annual profits on total assets is A $2,500 C $3,000 B $3,500 D $4,500 58 The firm's annual financing costs of the aggressive financing strategy is (M) A $21,175 C $24,475 B $26,075 D $22,775 Questions 62 thru 68 are based on the following information Flum Packages, Inc D 3:2 66 If the firm was to shift $7,000 of fixed assets to current assets, the firm's net working capital would _, the annual profits on total assets would _, and the risk of not being able to meet current obligations would _, respectively A increase; decrease; increase C increase; decrease; decrease B decrease; increase; decrease D decrease; increase; increase 67 If the firm was to shift $2,000 of current liabilities to long-term funds, the firm's net working capital would _, the annual cost of financing would _, and the risk of technical insolvency would _, respectively A decrease; decrease; increase C decrease; increase; decrease B increase; increase; decrease D increase; decrease; decrease 68 The firm would like to increase its current ratio This goal would be accomplished most profitably by A increasing current liabilities C increasing current assets B decreasing current liabilities D decreasing current assets CASH MANAGEMENT Cash Conversion Cycle 29 A firm has an average age of inventory of 60 days, an average collection period of 45 days, and an average payment period of 30 days The firm's cash conversion cycle is days (E) Page of 58 MANAGEMENT ADVISORY SERVICES A 15 B 45 WORKING CAPITAL FINANCE C 75 D 135 34 A firm has an average age of inventory of 101 days, an average collection period of 49 days, and an average payment period of 60 days The firm's cash conversion cycle is (E) A 150 days C 112 days B 90 days D days Gitman 37 A firm has an average age of inventory of 20 days, an average collection period of 30 days, and an average payment period of 60 days The firm's cash conversion cycle is _ days (E) A 70 C -10 B 50 D 110 Gitman 16 If the average age of inventory is 60 days, the average age of the accounts payable is 30 days, and the average age of accounts receivable is 45 days, the number of days in the cash flow cycle is (E) A 135 days C 75 days B 90 days D 105 days Gleim 17 If the average days of inventory is 90 days, the average age of accounts payable is 60 days, and the average age of accounts receivable is 65 days, the number of days in the cash flow cycle is (E) a 215 days c 95 days b 150 days d 85 days CMA 1284 1-20, RPCPA 1096 18 For the Cook County Company, the average age of accounts receivable is 60 days, the average age of accounts payable is 45 days, and the average age of inventory is 72 days Assuming a 360-day year, what is the length of the firm’s cash conversion cycle? (E) a 87 days d 48 days b 90 days e 66 days c 65 days Brigham 19 A growing company is assessing current working capital requirements An average of 58 days is required to convert raw materials into finished goods and to sell them Then an average of 32 days is required to collect on receivables If the average time the company takes to pay for its raw materials is 15 days after they are received, then the total cash conversion cycle for this company is (E) CMA EXAMINATION QUESTIONS A 11 days B 41 days Gitman C 75 days D 90 days CIA 0596 IV-53 20 Spartan Sporting Goods has $5 million in inventory and $2 million in accounts receivable Its average daily sales are $100,000 The company’s payables deferral period (accounts payable divided by daily purchases) is 30 days What is the length of the company’s cash conversion cycle? (E) a 100 days d 40 days b 60 days e 33 days c 50 days Brigham 31 A firm purchased raw materials on account and paid for them within 30 days The raw materials were used in manufacturing a finished good sold on account 100 days after the raw materials were purchased The customer paid for the finished good 60 days later The firm's cash conversion cycle is days (M) A 10 C 130 B 70 D 190 Gitman 87 A firm with a cash conversion cycle of 175 days can stretch its average payment period from 30 days to 45 days This will result in a(n) _ in the cash conversion cycle of _ days (M) A increase, 15 C increase, 45 B decrease, 15 D decrease, 45 Gitman 21 Porta Stadium Inc has annual sales of $40,000,000 and keeps average inventory of $10,000,000 On average, the firm has accounts receivable of $8,000,000 The firm buys all raw materials on credit, its trade credit terms are net 30 days, and it pays on time The firm’s managers are searching for ways to shorten the cash conversion cycle If sales can be maintained at existing levels but inventory can be lowered by $2,000,000 and accounts receivable lowered by $1,000,000, what will be the net change in the cash conversion cycle? Use a 360-day year (M) a +105 days d -27 days b -105 days e -3 days c +27 days Brigham 22 Bully Corporation purchases raw materials on July It converts the raw materials into inventory by September 30 However, Bully pays for the materials on July 20 On October 31, it sells the finished goods inventory Then, the firm collects cash from the sale month later on Page of 58 MANAGEMENT ADVISORY SERVICES WORKING CAPITAL FINANCE November 30 If this sequence accurately represents the average working capital cycle, what is the firm's cash conversion cycle in days? (D) A 92 days C 123 days B 133 days D 153 days Gleim a -40 days b -22 days c -13 days d +22 days e +40 days Brigham 26 23 You have recently been hired to improve the performance of Multiplex Corporation, which has been experiencing a severe cash shortage As one part of your analysis, you want to determine the firm’s cash conversion cycle Using the following information and a 360-day year, what is your estimate of the firm’s current cash conversion cycle? (M) Current inventory = $120,000 Annual sales = $600,000 Accounts receivable = $160,000 Accounts payable = $25,000 Total annual purchases = $360,000 Purchases credit terms: net 30 days Receivables credit terms: net 50 days a 49 days d 168 days b 193 days e 143 days c 100 days Brigham 24 Gaston Piston Corp has annual sales of $50,000,000 and maintains an average inventory level of $15,000,000 The average accounts receivable balance outstanding is $10,000,000 The company makes all purchases on credit and has always paid on the 30th day The company is now going to take full advantage of trade credit and pay its suppliers on the 40th day If sales can be maintained at existing levels but inventory can be lowered by $2,000,000 and accounts receivable lowered by $2,000,000, what will be the net change in the cash conversion cycle? (Assume there are 360 days in the year.) (M) a -14.4 days d -25.6 days b -18.8 days e -38.8 days c -28.8 days Brigham 25 Kolan Inc has annual sales of $36,500,000 ($100,000 a day on a 365-day basis) On average, the company has $12,000,000 in inventory and $8,000,000 in accounts receivable The company is looking for ways to shorten its cash conversion cycle, which is calculated on a 365-day basis Its CFO has proposed new policies that would result in a 20 percent reduction in both average inventories and accounts receivables The company anticipates that these policies will also reduce sales by 10 percent Accounts payable will remain unchanged What effect would these policies have on the company’s cash conversion cycle? (M) CMA EXAMINATION QUESTIONS Jordan Air Inc has average inventory of $1,000,000 Its estimated annual sales are 12 million and the firm estimates its receivables conversion period to be twice as long as its inventory conversion period The firm pays its trade credit on time; its terms are net 30 The firm wants to decrease its cash conversion cycle by 10 days It believes that it can reduce its average inventory to $900,000 Assume a 360-day year and that sales will not change By how much must the firm also reduce its accounts receivable to meet its goal of a 10-day reduction? (D) a $ 101,900 d $ 333,520 b $1,000,000 e $ c $ 233,333 Brigham 209.A firm has arranged for a lockbox system to reduce collection time of accounts receivable Currently the firm has an average collection period of 43 days, an average age of inventory of 50 days, and an average payment period of 10 days The lockbox system will reduce the average collection period by days by reducing processing, mail, and clearing float The firm's cash conversion cycle _ (E) A increases by days C will not change B decreases by days D is 93 days Annual Savings 81 A firm has annual operating outlays of $1,800,000 and a cash conversion cycle of 60 days If the firm currently pays 12 percent for negotiated financing and reduces its cash conversion cycle to 50 days, the annual savings is A $ 50,000 C $ 6,000 B $200,000 D $216,000 Gitman 82 A firm has a cash conversion cycle of 60 days Annual outlays are $12 million and the cost of negotiated financing is 12 percent If the firm reduces its average age of inventory by 10 days, the annual savings is A $104,000 C $ 28,800 B $144,000 D $40,000 Gitman Page of 58 MANAGEMENT ADVISORY SERVICES Cash Flow 27 FLF Corporation had income before taxes of $50,000 Included in the calculation of this amount was depreciation of $6,000, a charge of $7,000 for the amortization of bond discounts, and $5,000 for interest paid The estimated pretax cash flow for the period is (M) A $50,000 C $37,000 B $57,000 D $63,000 Gleim 28 RLF Corporation had income before taxes of $60,000 for the year 1991 Included in this amount was depreciation of $5,000, a charge of $6,000 for amortization of bond discounts, and $4,000 for interest expense The estimated cash flow for the period is (M) a $60,000 c $49,000 b $66,000 d $71,000 CMA 0692 1-26 WORKING CAPITAL FINANCE Suppose that the interest rate on Treasury bills is 6%, and every sale of bills costs $60 You pay out cash at a rate of $800,000 a year According to Baumol's model of cash balances, how many times a year should you sell bills? A 20 C 50 B 35 D 15 B&M Q Suppose that the interest rate on Treasury bills is 6%, and every sale of bills costs $60 You pay out cash at a rate of $800,000 a year According to Baumol's model of cash balances, what is Q? A $17,376 D $50,000 B $20,000 E $40,000 C $10,000 B&M 29 Shown below is a forecast of sales for Cooper Inc for the first months of the year (all amounts are in thousands of dollars) January February March April Cash sales $ 15 $ 24 $18 $14 Sales on credit 100 120 90 70 On average, 50% of credit sales are paid for in the month of sale, 30% in the month following the sale, and the remainder is paid months after the month of sale Assuming there are no bad debts, the expected cash inflow for Cooper in March is (M) A $138,000 C $119,000 B $122,000 D $108,000 CMA 1295 1-8 Baumol’s Model of Cash Balances Number of Conversions Suppose that the interest rate on Treasury bills is 6%, and every sale of bills costs $20 You pay out cash at a rate of $400,000 a month According to Baumol's model of cash balances, how many times a month should you sell bills? A 30 C B 20 D B&M Suppose that the interest rate on Treasury bills is 4%, and every sale of bills costs $40 You pay out cash at a rate of $1,000,000 a quarter According to Baumol's model of cash balances, how many times a quarter should you sell bills? (Approximately.) A 20 C 12 B 22 D 11 B&M CMA EXAMINATION QUESTIONS Suppose that the interest rate on Treasury bills is 6%, and every sale of bills costs $20 You pay out cash at a rate of $400,000 a month According to Baumol's model of cash balances, what is Q? A $16,000 D $43,000 B $24,000 E $57,000 C $31,000 B&M Float Availability Float 18 On an average day, a company writes checks totaling $1,500 These checks take days to clear The company receives checks totaling $1,800 These checks take days to clear The cost of debt is 9% What is the firm's availability float? A $10,500 C $1,800 B $7,200 D None of the above B&M Disbursement Float 17 On an average day, a company writes checks totaling $1,500 These checks take days to clear The company receives checks totaling $1,800 These checks take days to clear The cost of debt is 9% What is the firm's disbursement float? A $10,500 C $1,800 B $1,500 D None of the above B&M Page of 58 MANAGEMENT ADVISORY SERVICES 20 H Pottamus, Inc., has $2 million on deposit with the bank It now writes checks for $100,000 and $200,000 and deposits a check for $80,000 Two weeks later it learns that the $200,000 check and $80,000 check have cleared What is the company's disbursement float? A $300,000 C $100,000 B $220,000 D -$100,000 B&M Net Float 19 On an average day, a company writes checks totaling $1,500 These checks take days to clear The company receives checks totaling $1,800 These checks take days to clear The cost of debt is 9% What is the firm's net float? A $300 C $2,100 B $3,300 D $1,200 B&M WORKING CAPITAL FINANCE A realized an annual loss of $120,000 B realized an annual savings of $120,000 C increased its cash cycle D decreased its cash turnover Gitman Opportunity Cost 33 What is the opportunity cost of keeping a cash balance of $2 million, if the daily interest rate is 0.02% and the average transaction cost of investing money overnight is $50? (E) A $50 C $400 B $350 D $40,000 Gleim Questions and are based on the following information CIA 0595 IV-45 & 46 A company has a 10% cost of borrowing and incurs fixed costs of $500 for obtaining a loan It has stable, predictable cash flows, and the estimated total amount of net new cash needed for transactions for the year is $175,000 The company does not hold safety stocks of cash 30 Assume that each day a company writes and receives checks totaling $10,000 If it takes days for the checks to clear and be deducted from the company's account, and only days for the deposits to clear, what is the float? (E) A $10,000 C $(10,000) B $0 D $50,000 CMA 0694 1-24 31 Average daily collection of checks for a firm is $40,000 The firm also writes on the average $35,000 of checks daily If the collection period for checks is days, calculate the net float (E) A $25,000 C $175,000 B $40,000 D $200,000 Gleim 32 Jumpdisk Company writes checks averaging $15,000 a day, and it takes five days for these checks to clear The firm also receives checks in the amount of $17,000 per day, but the firm loses three days while its receipts are being deposited and cleared What is the firm’s net float in dollars? (E) a $126,000 d $ 24,000 b $ 75,000 e $ 16,000 c $ 32,000 Brigham Annual Savings 12 As part of a union negotiation agreement, the United Clerical Workers Union conceded to be paid every two weeks instead of every week A major firm employing hundreds of clerical workers had a weekly payroll of $1,000,000 and the cost of short-term funds was 12 percent The effect of this concession was to delay clearing time by one week Due to the concession, the firm CMA EXAMINATION QUESTIONS 34 When the average cash balance of the company is higher, the the cash balance is List A List B A Opportunity cost of holding Higher B Total transactions costs associated with obtaining Higher C Opportunity cost of holding Lower D Total costs of holding Lower 35 If the average cash balance for the company during the year is $20,916.50, the opportunity cost of holding cash for the year will be A $2,091.65 C $8,750.00 B $4,183.30 D $17,500.00 Lock Box Service Increase in Average Cash Balance 36 CMR is a retail mail order firm that currently uses a central collection system that requires all checks to be sent to its Boston headquarters An average of days is required for mailed checks to be received, days for CMR to process them and 1½ days for the checks to clear through its bank A proposed lockbox system would reduce the mail and process time to days and the check clearing time to day CMR has an average daily collection of $100,000 If CMR should adopt the lockbox system, its average cash balance would increase by (E) a $250,000 c $650,000 b $400,000 d $800,000 CMA 1286 1-30 Page of 58 MANAGEMENT ADVISORY SERVICES WORKING CAPITAL FINANCE 37 DLF is a retail mail order firm that currently uses a central collection system that requires all checks to be sent to its Boston headquarters An average of days is required for mailed checks to be received, days for DLF to process them, and days for the checks to clear through its bank A proposed lockbox system would reduce the mailing and processing time to days and the check clearing time to day DLF has an average daily collection of $150,000 If DLF adopts the lockbox system, its average cash balance will increase by (E) A $1,200,000 C $600,000 B $750,000 D $450,000 Gleim Daily Income (Loss) 24 Assume that the average number of daily payments to a lock-box is 200, the average size of the payment is $1,000, the rate of interest per day is 0.02% (i.e., 0.0002), the savings in mail time is days, and the savings in processing time is day What is the daily return from operating the lock-box? A $80 C $120 B $100 D $130 B&M Increase in Annual Income (Loss) 38 What are the expected annual savings from a lockbox system that collects 200 checks per day averaging $500 each, and reduces mailing and processing times by 2.0 and 0.5 days, respectively, if the annual interest rate is 6%? (E) A $250,000 C $6,000 B $12,000 D $15,000 Gleim 39 A firm has daily cash receipts of $300,000 A bank has offered to provide a lockbox service that will reduce the collection time by days The bank requires a monthly fee of $2,000 for providing this service If money market rates are expected to average 6% during the year, the additional annual income (loss) of using the lockbox service is (E) A ($24,000) C $30,000 B $12,000 D $54,000 Gleim 40 Foster Inc is considering implementing a lock box collection system at a cost of $80,000 per year Annual sales are $90 million, and the lockbox system will reduce collection time by days If Foster can invest funds at 8%, should it use the lockbox system? Assume a 360-day year (E) a Yes, producing savings of $140,000 per year b Yes, producing savings of $60,000 per year c No, producing a loss of $20,000 per year CMA EXAMINATION QUESTIONS d No, producing a loss of $60,000 per year CMA 1295 1-14 41 Cross Collectibles currently fills mail orders from all over the U S and receipts come in to headquarters in Little Rock, Arkansas The firm’s average accounts receivable (A/R) is $2.5 million and is financed by a bank loan with 11 percent annual interest Cross is considering a regional lockbox system to speed up collections that it believes will reduce A/R by 20 percent The annual cost of the system is $15,000 What is the estimated net annual savings to the firm from implementing the lockbox system? (M) a $500,000 d $ 55,000 b $ 30,000 e $ 40,000 c $ 60,000 Brigham 42 A company has daily cash receipts of $150,000 The treasurer of the company has investigated a lock box service whereby the bank that offers this service will reduce the company’s collection time by four days at a monthly fee of $2,500 If money market rates average 4% during the year, the additional annual income (loss) from using the lock box service would be (E) a $6,000 c $12,000 b $(6,000) d $(12,000) CMA 0694 1-19 43 A banker has offered to set up and operate a lock box system for your company Details are given below Estimate the annual savings Average number of daily payments 325 Average size of payments $1,250 Daily interest rate 0.021% Saving in mailing time 1.3 days Saving in processing time 0.9 days Bank charges $0.30 Assume 250 processing days per year (M) A $3,273 C $23,500 B $22,675 D $47,000 Gleim 44 Cleveland Masks and Costumes Inc (CMC) has a majority of its customers located in the states of California and Nevada Keystone National Bank, a major west coast bank, has agreed to provide a lockbox system to CMC at a fixed fee of $50,000 per year and a variable fee of $0.50 for each payment processed by the bank On average, CMC receives 50 payments per day, each averaging $20,000 With the lockbox system, the company's collection float will decrease by days The annual interest rate on money market securities is 6% If Page of 58 MANAGEMENT ADVISORY SERVICES CMC makes use of the lockbox system, what would be the net benefit to the company? Use 365 days per year (M) A $59,125 C $50,000 B $60,875 D $120,000 CMA Samp Q1-6 Optimal Lock-Box Alternative 45 Newman Products has received proposals from several banks to establish a lockbox system to speed up receipts Newman receives an average of 700 checks per day averaging $1,800 each, and its cost of short-term funds is 7% per year Assuming that all proposals will produce equivalent processing results and using a 360-day year, which one of the following proposals is optimal for Newman? (M) CMA 0697 1-13 a A $0.50 fee per check c A fee of 0.03% of the amount collected b A flat fee of $125,000 per year d A compensating balance of $1,750,000 46 A firm has daily cash receipts of $300,000 and is interested in acquiring a lockbox service in order to reduce collection time  Bank 1's lockbox service costs $3,000 per month and will reduce collection time by days  Bank 2's lockbox service costs $5,000 per month and will reduce collection time by days  Bank 3's lockbox service costs $500 per month and will reduce collection time by day  Bank 4's lockbox service costs $1,000 per month and will reduce collection time by days If money market rates are expected to average 6% during the year, and the firm wishes to maximize income, which bank should the firm choose? (M) A Bank C Bank B Bank D Bank Gleim Other Cash Management Systems Change in Profit (Loss) * QRS makes large cash payments averaging P17,000 daily The company changed from using checks to sight drafts which will permit it to hold unto its cash for one extra day If QRS can use the extra cash to earn 14% annually, what annual peso return will it earn? (E) a P652.10 c P6.52 b P6,521.00 d P2,380 RPCPA 1097 47 What is the benefit for a firm with daily sales of $15,000 to be able to reduce the collection period by days, given an 8% annual opportunity cost of funds? (M) CMA EXAMINATION QUESTIONS WORKING CAPITAL FINANCE A $2,400 annual benefit B $1,200 annual benefit C $600 annual benefit D $7,500 annual benefit Gleim 48 A firm has daily cash receipts of $100,000 and collection time of days A bank has offered to decrease the collection time on the firm’s deposits by two days for a monthly fee of $500 If money market rates are expected to average 6% during the year, the net annual benefit loss) from having this service is (M) a $3,000 c $0 b $12,000 d $6,000 CMA 0696 1-12 49 A firm has daily cash receipts of $200,000 A commercial bank has offered to reduce the collection time by days The bank requires a monthly fee of $4,000 for providing this service If money market rates will average 12% during the year, the additional annual income (loss) of having the service is (M) A $(24,000) C $66,240 B $24,000 D $68,000 CMA 0683 1-7 50 A firm has daily cash receipts of $300,000 A commercial bank has offered to reduce the collection time by days The bank requires a monthly fee of $3,000 for providing this service If the money market rates will average 11% during the year, the annual pretax income (loss) from using the service is (M) A $(30,000) C $66,000 B $30,000 D $63,000 Gleim Point of Indifference 51 Average daily cash outflows are $3 million for Farms Inc A new cash management system can add days to the disbursement schedule Assuming Farms earns 10% on excess funds, how much should the firm be willing to pay per year for the cash management system (E) a $6,000,000 c $1,500,000 b $3,000,000 d $600,000 CMA 1295 1-3 52 Troy Toys is a retailer operating in several cities Its individual store managers deposit daily collections at a local bank in a noninterest-bearing checking account Twice per week, the local bank issues a depository transfer check (DTC) to the central bank at headquarters The controller of the company is considering using a wire transfer instead The additional cost of each transfer would be $25; collections would be accelerated by two days; and an annual interest rate paid by the central bank is 7.2% (0.02% per day) At what amount of dollars Page 10 of 58 DISCUSSION: (B) If sals are $4,000 per day, and customers pay in 30 days, 30 days of sales are outstanding, or $120,000 Whether customers pay by credit card or cash, collection requires 30 days Answer (A) is incorrect because $4,000 is only one day’s seales Answer (C) is incorrect because invoices are outstanding for 30 days, not 12 days Answer (D) is incorrect because $54,000 is based on the 45% of collections via credit card 67 Accounts receivable Answer: a Diff: M First solve for current annual sales using the DSO equation as follows: 48 = $1,000,000/(Sales/360) to find annual sales equal to $7,500,000 If sales fall by 10%, the new sales level will be $7,500,000(0.9) = $6,750,000 Again, using the DSO equation, solve for the new accounts receivable figure as follows: 32 = AR/($6,750,000/360) or AR = $600,000 68 Answer (C) is correct Days' sales outstanding is the average collection period Assuming that all sales are on credit, that one-third of the customers take advantage of the 5% cash discount and pay on day 10, and that the remaining two-thirds of the customers pay on day 20, the days' sales outstanding is 17 days [(.33 x 10 days) + (.67 x 20 days)] Answer (A) is incorrect because 13 days includes only the days' sales outstanding for customers that not take the cash discount Answer (B) is incorrect because 15 days uses five days until payment for the customers that take the cash discount Answer (D) is incorrect because 20 days equals the 20-day collection period for customers not taking the cash discount 69 REQUIRED: The projected days’ sales outstanding DISCUSSION: (C) Given than 40% of sales will be collected on the 15th day, 40% on the 30th day, and 20% on the 45th day, the days’ sales outstanding can be determined by weighting the collection period for each group of receivables by its collection percentage Hence, the projected days’ sales outstanding equal 27 days [(40% x 15) + (40% x 30) + 20% x 45)] Answer (A) is incorrect because average receivables are outstanding for much more than 20 days Answer (B) is incorrect because 24 days assumes 40% of receivables are collected after 15 days and 60% after 30 days Answer (D) is incorrect because more receivables are collected on the 15th day than on the 45th day; thus, the average must be less than 30 days 70 Answer (A) is correct Credit sales amounted to $540,000 (60% x $900,000) Dividing $540,000 by 4.5 produces an average receivable balance of $120,000 Dividing 365 days by the turnover of 4.5 results in an average collection period of 81 days Answer (B) is incorrect because the collection period is 81 days Answer (C) is incorrect because the collection period is 81 days and the average receivables is too low Answer (D) is incorrect because it includes cash sales as well as credit sales in receivables 71 Answer (C) is correct The average collection period is the average time it takes to receive payment from customers because one-half of the customers will pay on day 10 and half will pay on day 30, the average collection period is 20 days [.5(10 days) + 5(30 days)] Answer (A) is incorrect because 10 days assumes all customers take the discount Answer (B) is incorrect because 15 days assumes half of the customers pay on day 30 but ignores the remaining half of the customers who pay on day 10 Answer (D) is incorrect because 30 days assumes all customers pay on day 30 72 Answer (D) is correct The expected average accounts receivable balance equals the average collection period times the credit sales per day Thus, the average accounts receivable balance is 45,205.48 {[(10,000 units sold on credit x 66 price) ÷ 365 days] x 25 days} The foregoing calculation assumes that receivables are recorded at their gross amounts Answer (A) is incorrect because 684.93 is based on annual credit sales of 10,000 Answer (B) is incorrect because 1,808.22 equals the credit sales per day Answer (C) is incorrect because 27,123.30 is based on a 15-day average collection period 73 Answer (B) is correct Seasonal dating is a credit policy under which a customer may make purchases early but payment is not due until the customer's retail selling season begins When seasonal dating is used, the selling company incurs higher credit costs because customers have longer to pay Thus, reduced credit costs are not an advantage of seasonal dating Answer (A) is incorrect because reduced storage costs result when customers buy earlier Answer (C) is incorrect because attractive credit terms result when customers can buy earlier and not pay until their selling season begins Answer (D) is incorrect because, when customers buy earlier, uncertainty about sales volume is reduced 74 Answer (C) is correct Assuming that all of the customers take advantage of seasonal dating and the discount, the collections on or before January 11 will be the number of units sold, times the unit selling price, times minus the discount percentage, or $6,860 [(700 units x $10)(1 - 02)] Answer (A) is incorrect because all of the customers are taking the discount and will pay on or before January 11 Answer (B) is incorrect because $6,370 does not include the collection of the revenue from units sold on January Answer (D) is incorrect because $7,000 does not consider the 2% discount 75 Answer (D) is correct Terms of 1/10, net 60 mean that a buyer can save 1% of the purchase price by paying 50 days early In essence, not taking the discount results in the buyer's borrowing 99% of the invoice price for 50 days at a total interest charge of 1% of the invoice price Because a year has 7.3 50-day periods (365 ÷ 50), the credit terms 1/10, net 60 yield an effective annualized interest charge of approximately 7.37% [(1% ÷ 99%) x 7.3] If the prime rate were higher than 7.37%, the buyer would prefer to borrow from the vendor (i.e., not pay within the discount period) rather than from a bank Consequently, an 8% prime rate could cause the vendor's receivables to increase Answer (A) is incorrect because the prime rate must be greater than 7.37% to make the company's terms preferable to those of a bank Answer (B) is incorrect because the prime rate must be greater than 7.37% to make the company's terms preferable to those of a bank Answer (C) is incorrect because the prime rate must be greater than 7.37% to make the company's terms preferable to those of a bank 76 Answer (B) is correct This firm's daily credit sales equal $4,000 (40% x 1,000 units x $10 price) Hence, reducing the accounts receivable collection period by 10 days will decrease the balance by $40,000 (10 days x $4,000) Answer (A) is incorrect because a $4,000 decrease assumes a 1-day reduction Answer (C) is incorrect because $240,000 is the total balance of accounts receivable under the new policy Answer (D) is incorrect because $280,000 is the total balance of accounts receivable under the old policy 77 REQUIRED: The dollar impact on accounts receivable of a change in credit policy DISCUSSION: (C) If sales are $50 million, 70% of which are on credit, total credit sales will be $35 million The receivables turnover equals 4.8 times per year (360 days ÷ 75-day collection period) Receivables turnover equals net credit sales divided by average receivables Accordingly, average receivables equal $7,291,667 ($35,0000 ÷ 4.8) Under the new policy, sales will be $47.5 million (95% x $50,000,000), and credit sales will be $28.5 million (60% x $47,500,000) The collection period will be reduced to 50 days, resulting in a turnover of 7.2 times per year (360 ÷ 50) The average receivables balance will therefore be $3,958,333 ($28,500,000 ÷ 7.2), a reduction of $3,333,334 ($7291,667 - $3,958,333) Answers (A) and (B) are incorrect because the decrease will be $3,333,334 Answer (D) is incorrect because receivables will decrease 78 REQUIRED: The expected increase in the average accounts receivable balance as a result of relaxed credit standards DISCUSSION: (D) Of the $40,500,000 of sales, 80% are expected to be on credit, a total of $32,400,000 Average daily credit sales are therefore $90,000 ($32,400,000 ÷ 360) If 30 days of sales are outstanding at any one time, the average balance in accounts receivable is $2,700,000 If credit sales increase by 20%, the aforementioned $90,000 average of daily sales will increase to $108,000 (120% x $90,000) For an average collection period of 40 days, the average accounts receivable balance will be $4,320,000 (40 x $108,000) Hence, the expected increase in the balance is $1,620,000 ($4,320,000 - $2,700,000) Answer (A) is incorrect because $540,000 would have been the increase if the average balance outstanding remains at 30 days of sales Answer (B) is incorrect because $2,700,000 is the average amount in accounts receivable before any change in credit terms Answer (C) is incorrect because $900,000 would have been the increase based solely on the increased number of days outstanding, assuming no increase in sales 79 Answer (B) is correct Of the $50,000,000 of sales, 75% are expected to be on credit, a total of $37,500,000 Average daily credit sales are therefore $104,167 ($37,500,000 ÷ 360 days) If the average collection period is 20 days, the average balance in accounts receivable is $2,083,340 (20 x $104,167) If credit sales increase by 25%, average daily sales will increase to $130,209 (125% x $104,167) For an average collection period of 30 days, the average accounts receivable balance will be $3,906,270 (30 x $130,209) Hence, the expected increase in the balance is $1,822,930 ($3,906,270 - $2,083,340) Answer (A) is incorrect because $520,840 is based on a 20-day collection period and a 25% increase in credit sales Answer (C) is incorrect because $2,083,340 is the average accounts receivable balance before any change in credit terms Answer (D) is incorrect because $3,906,270 is the average accounts receivable balance given the change in credit terms 80 Accounts receivable increase Answer: b Diff: M R DSO = $41,096/($250,000/365) = 60 days New A/R = [($250,000)(1.5)/(365)](60)(1.5) = $92,466 Hence, increase in receivables = $92,466 - $41,096 = $51,370 81 Accounts receivable Answer: a Diff: M R First solve for current annual sales using the DSO equation as follows: 50 = $1,000,000/(Sales/365) to find annual sales equal to $7,300,000 If sales fall by 10%, the new sales level will be $7,300,000(0.9) = $6,570,000 Again, using the DSO equation, solve for the new accounts receivable figure as follows: 32 = AR/($6,570,000/365) or AR = $576,000 82 REQUIRED: The expected discounts taken in the coming year if a 2% discount is allowed for early payment DISCUSSION: (A) If 90% of the $24,000,000 of sales are on credit, $21,600,000 of sales will be subject to the discount If 50% of the credit customers take the discount, discounts will be taken on sales of $10,800,000 The expected discount will be $216,000 (2$ of $10,800,000) Answer (B) is incorrect because $432,000 would have been the discount on all of the credit sales Answer (C) is incorrect because only 90% of the sales are on credit; $240,000 would be correct only if a cash discount were allowed on cash sales as well as credit sales Answer (D) is incorrect because $480,00 is based on the assumption that all sales will be discounted 83 Answer (A) is correct If 80% of the $20,000,000 of sales are on credit, $16,000,000 of sales will be subject to the discount If discounts are taken on 60% of credit sales, the expected discount will be $288,000 [3% x (60% x $16,000,000)] Answer (B) is incorrect because $480,000 is the potential discount on all credit sales Answer (C) is incorrect because $360,000 assumes all sales are on credit and that 60% of discounts are taken Answer (D) is incorrect because $600,000 assumes all sales are on credit and that 100% of the discounts are taken 84 Answer (A) is correct The cost of carrying receivables equals average receivables times the variable cost ratio times the cost of money Under the old policy, average daily sales are $12,778 ($4,600,000 ÷ 360 days) Given a 30-day average collection period, the average receivables balance is $383,340 ($12,778 x 30 days) Under the new policy, average daily sales are $13,778 ($4,960,000 ÷ 360 days), and the average receivables balance is $482,230 ($13,778 x 35 days) Hence, the average balance is $98,890 higher under the new policy Because the company's incremental (variable) costs are 50% of sales, the additional investment is $49,445 (50% x $98,890) The interest rate, or required rate of return, is 11% Thus, the incremental pretax carrying cost is $5,439 (11% x $49,445) Answer (B) is incorrect because $10,878 fails to adjust for the proportion of incremental costs included in the additional receivables Answer (C) is incorrect because $13,778 is the average daily sales under the new policy Answer (D) is incorrect because $98,890 is the amount of the additional receivables 85 Answer (A) is correct The first step is to determine the average investment in receivables under each policy Under the old policy, average daily sales are $10,000 ($3,600,000/360 days) Given a 30-day average collection period, the average receivables balance is $300,000 ($10,000 x 30 days) Under the new policy, average daily sales are $11,000 ($3,960,000/360 days), and the average receivables balance is $396,000 ($11,000 x 36 days) Hence, the average balance is $96,000 higher under the new policy Because the company's incremental (variable) costs are 60% of sales, the extra investment is only $57,600 (60% x $96,000) The interest rate, or required rate of return, is 10% Thus, the incremental carrying cost is $5,760 (10% x $57,600) Answer (B) is incorrect because $9,600 equals the 10% required rate of return times the $96,000 differential between the average receivables balances Answer (C) is incorrect because the differential between the average receivables balances is $96,000, not $136,000 Answer (D) is incorrect because $960 assumes a 10% variable cost ratio 86 REQUIRED: The annual benefit or loss resulting from a change in credit terms DISCUSSION: The incremental sales will produce an increased contribution margin of $144,000 (20% x $720,000) However, that amount must be offset by the cost of funds invested in receivables The variable costs associated with the incremental sales are $576,000 (80% of $720,000) Given a 75-day credit period, the average investment in receivables equals $120,000 ($576,000 x (75 ÷ 360) Accordingly, the cost of the investment in additional receivables is $24,000 (20% opportunity cost x $120,000), and the net benefit of the planned change in credit terms is $120,000 ($144,000 - $24,000) Answer (A) is incorrect because the company benefits from the change in credit terms Answer (B) is incorrect because $28,800 results from multiplying the contribution margin by the 20% interest rate Answer (C) is incorrect because $144,000 overlooks the costs created by having funds invested in receivables for 75 days, 87 REQUIRED: The increase in after-tax profit as a result of an increase in sales DISCUSSION: (B) The company’s manufacturing and selling costs exclusive of bad debts equal 70% of bad debts Hence, the gross profit on the $100,000 increase in sales will be $30,000 (30% x $100,000) Assuming $15,000 of bad debts and $5,000 of collection expense, the increase in pretax income will be $10,000 ($30,000 - $20,000) Consequently, after0tax income will increase by $6,000 [$10,000 – (40% x $10,000) Answers (A), (C) and (D) are incorrect because after-tax income will increase by $6,000 88 Answer (B) is correct The company's manufacturing and selling costs exclusive of bad debts equal 75% of sales Hence, the gross profit on the $150,000 increase in sales will be $37,500 (25% x $150,000) Assuming $24,000 (16% x $150,000) of bad debts and $6,000 (4% x $150,000) of collection expense, the increase in pretax income will be $7,500 ($37,500 - $30,000) Consequently, after-tax income will increase by $4,650 [$7,500 - (38% x $7,500)] Answer (A) is incorrect because $2,850 is based on a 62% tax rate, the complement of the actual tax rate Answer (C) is incorrect because $7,500 is the pretax income Answer (D) is incorrect because $8,370 omits collection costs from the calculation 89 Credit policy and ROE Answer: c Diff: M R Use the DSO formula to calculate accounts receivable under the new policy as 36 = AR/($730,000/365) or AR = $72,000 Thus, $125,000 - $72,000 = $53,000 is the cash freed up by reducing DSO to 36 days Retiring $53,000 of long-term debt leaves $247,000 in long-term debt Given a 10% interest rate, interest expense is now $247,000(0.1) = $24,700 Thus, EBT = EBIT - Interest = $70,000 - $24,700 = $45,300 Net income is $45,300(1 0.3) = $31,710 Thus, ROE = $31,710/$200,000 = 15.86% 90 Answer (C) is correct This problem can be solved using incremental analysis Since incremental profits will be 20% of sales, and bad debt expense (the only incremental expense) will be less than 20%, then both options will increase profits Answer (A) is incorrect because, under both new policies, the incremental costs will be less than incremental revenues Thus, profits will be increased under either Policy A or B Answer (B) is incorrect because the 15% increase in cost will be less than the 20% profit margin Answer (D) is incorrect because all policies will result in different profit levels 91 REQUIRED: The annual percentage cost of financing under a factoring agreement DISCUSSION: (D) In an average month, the company will receive $80,000 at the time the receivables are sent to the factor Over a year’s time, the interest on this average advance of $80,000 would be $8,000 at 10% interest In addition, the factor will charge a 2% factor fee, or $24,000 ($100,000 x 12 x 0.02) over the course of a year However, this $24,000 fee is offset by the $18,000 savings in collection expenses, producing a net outlay of only $6,000 Adding the $6,000 to the $8,000 of interest produces an annual net cost of $14,000 Dividing the $14,000 cost by the $80,000 of advanced funds results in a cost of 17.5% Answer (A) is incorrect because 10% overlooks the factor fee Answer (B) is incorrect because 12% overlooks the fact that the 2% fee recurs every month Answer (C) is incorrect because 14% miscalculates the factor fee and the savings from reduced collection costs 92 Answer (C) is correct The factor will withhold $6,000 (6% x $100,000) as a reserve against returns and allowances and $1,400 (1.4% x $100,000) as a commission The remaining $92,600 will be reduced by interest at the rate of 15% annually The interest charge should be $2,315, assuming a 360-day year [($92,600 x 15) ÷ (60day payment period ÷ 360 days)] The proceeds to be received by the seller equal $90,285 ($92,600 - $2,315) Answer (A) is incorrect because $92,600 ignores interest Answer (B) is incorrect because $96,135 fails to deduct the 6% reserve Answer (D) is incorrect because $85,000 assumes that the only amount withheld is a full year's interest on $100,000 93 Answer (D) is correct The calculation of the cash that the company will initially receive from factoring $100,000 in receivables is as follows: Amount of receivables$100,000Minus 6% reserve(6,000)Minus 1% factor fee (1,000)Amount accruing to the company$93,000Minus: 10% x(90/360) x $93,000(2,325)Cash to be received$90,675Answer (A) is incorrect because the amount received is also reduced by the interest charged Answer (B) is incorrect because $90,000 assumes the only deduction is for 10% interest for 360 days Answer (C) is incorrect because the interest charged will be for 90 days, not a full year 94 Answer (C) is correct The factor will hold out $8,800 (8% x $110,000) as a reserve against returns and allowances and $1,650 (1.5% x 110,000) as a commission That leaves $99,550 to be advanced to the seller However, interest at the rate of 16% annually is also to be withheld For 60 days that interest would amount to approximately $2,655 (assuming a 360-day year) The proceeds to be given to the seller equal $96,895 ($99,550 $2,655) Answer (A) is incorrect because $81,950 equals $99,550 minus 16% of $110,000 Answer (B) is incorrect because $83,630 is a nonsense answer Answer (D) is incorrect because $99,550 equals the amount received by the firm plus the interest charge 95 Answer (A) is correct Company A will withhold $6,000 (6% x $100,000) as a reserve against returns and allowances and $1,400 (1.4% x $100,000) as a commission The remaining $92,600 will be reduced by interest at the rate of 15% annually The interest charge will be $2,315, assuming a 360-day year [($92,600 x 15) ÷ (60-day payment period ÷ 360 days)] The proceeds to be received by Gatsby equal $90,285 ($92,600 - $2,315) Answer (B) is incorrect because Company B will produce proceeds of only $89,964 Answer (C) is incorrect because Company C will produce proceeds of only $90,190 Answer (D) is incorrect because Company D will produce proceeds of only $90,241 96 Answer (B) is correct With 3,000 accounts, 2,700 (90%) would be categorized as prompt paying and 300 as slow paying Multiplying 22% times the 300 slow-paying accounts results in 66 defaults Multiplying 5% times the 2,700 prompt-paying accounts results in an additional 135 defaults, for a total of 201 Answer (A) is incorrect because 795 uses 22% on all accounts, not just 300 of them Answer (C) is incorrect because 135 is only the defaults on the prompt-paying accounts Answer (D) is incorrect because 66 is only the defaults on the slow-paying accounts 97 Answer (B) is correct The firm will earn $100 of gross profit ($1,200 - $1,100) from each customer who pays but will lose $1,100 on each default The income statement for the 300 slow-pay customers would appear as follows: Sales (300 x 1,200)$360,000CGS (300 x 1,100) 330,000Gross profit30,000- Bad debt expense (66 x 1,200) (79,200)Net loss $(49,200)Dividing $49,200 by the 300 accounts produces an average loss of $164 Answer (A) is incorrect because $100 is the potential profit if there are no defaults Answer (C) is incorrect because $220 is the actual average loss per bad account but does not consider the $100 profit from the 78% who pay Answer (D) is incorrect because $264 is the write-off amount for each bad debt 98 Answer (C) is correct This can be solved algebraically with x being average revenue: 300x - $330,000 - 66x = 234x = $330,000 x = 1410.26 If average revenues were at least $1,410.26, the income statement would appear as follows: Sales (300 @ $1,410.26) $423,078.00CGS 330,000.00Gross profit93,078.00Bad debt expense (66 x $1,410.26) 93,077.16Net income$ 84Alternatively, another way of addressing the problem is to determine the amount of gross profit that will have to be generated by the 234 paying customers to cover the $1,100 loss on each of 66 defaulting customers Multiplying 66 x $1,100 produces a loss of $72,600 Dividing the $72,600 loss by the 234 paying customers results in $310.26 of gross profit that must be generated from every paying customer just to break even Adding the $1,100 cost to the $310.26 produces a revenue of $1,410.26 Answer (A) is incorrect because increasing the revenue by the amount of the average loss per account is insufficient because the additional $164 will be collected from only 234 customers, not all 300 Answer (B) is incorrect because a price of $1,389.74 will still result in an overall loss Answer (D) is incorrect because the $310.26 gross profit is added to cost, not revenue 99 Daily purchases = $720,000 = $2,000 360 Free trade credit = $2,000 × 15 = $30,000 100 Phranklin’s net purchases are $800,000 x (1 - 0.02) = $784,000 Purchases per day are $784,000/360 = $2,177.78 Total trade credit is 40 x $2,177.78 = $87,111.20 Free trade credit is 15 x $2,177.78 = $32,666.70 Thus, costly trade credit, assuming discounts are taken, is $87,111.20 - $32,666.70 = $54,444.50 If discounts are not taken, then the maximum amount of costly trade credit is $87,111.20 101 Approximate percentage cost = Accounts payable = 360 × = 0.2099 = 21% 98 35 $157,500 = $750,000 0.21    360      96    102 Nominal percentage cost =  = 3.00 = 300% 103 Answer (D) is correct Assume that the gross amount of an invoice is $1,000 With a 2% discount, the buyer will pay only $980 on the tenth day Thus, the seller is forgoing $20 to receive payment 20 days sooner than would otherwise be required The 20-day interest rate is 0204 ($20/$980) The number of 20-day periods in a year is 18 (360/20) If the interest rate is 2.04% for each 20-day period, the annual interest rate (rounded to the nearest tenth) is 36.7% (18 x 2.04%) Answer (A) is incorrect because the annual interest cost is 36.7% Answer (B) is incorrect because the annual interest cost is 36.7% Answer (C) is incorrect because the annual interest cost is 36.7% 104 Answer (D) is correct Assume that the gross amount of an invoice is $1,000 Given a 3% discount, the buyer will pay $970 on the tenth day Thus, the seller is forgoing $30 to receive payment 20 days sooner than would otherwise be required The 20-day interest rate (rounded) is 3.09% ($30 ÷ $970) The number of 20-day periods in a year is 18 (360 ÷ 20) If the interest rate is 3.09% for each 20-day period, the annual interest rate (rounded) is 55.6% (18 x 3.09) Answer (A) is incorrect because 36.7% assumes terms of 2/10, net 30 Answer (B) is incorrect because 24.5% assumes terms of 2/10, net 40 Answer (C) is incorrect because 37.1% assumes terms of 3/10, net 40 105 Answer (D) is correct Assume that an invoice is due in 45 days However, the seller allows a 2% cash discount if the invoice is paid within 15 days Given early payment, the seller will receive their money at least 30 days (45 15) sooner than the contract requires However, the seller is effectively paying 2% of the invoice price to receive the money 30 days early The approximate annual interest cost is 24% because a 360-day year contains 12 periods of 30 days each Answer (A) is incorrect because the 2% savings is for 30 days only; the annualized interest rate paid for receiving the money early is about 24% Answer (B) is incorrect because 16% assumes eight discount periods of 45 days each in a year rather than 12 Answer (C) is incorrect because 48% assumes 15-day discount periods 106 Answer (C) is correct The buyer could satisfy the $100 obligation by paying $98 on the 10th day By choosing to wait until the 40th day, the buyer is effectively paying a $2 interest charge for the use of $98 for 30 days (40-day credit period - 10-day discount period) The interest rate on what is essentially a 30-day loan is 2.04081% ($2 ÷ $98) Extrapolating this 30-day rate to a yearly rate involves multiplying by the number of periods in a year Thus, the effective annual rate is about 24.49% [2.04081% x (360 ÷ 30 days)] Answer (A) is incorrect because 2% is the discount rate Answer (B) is incorrect because 18.36% is based on the 40-day credit period Answer (D) is incorrect because 36.72% is based on a 20-day credit period 107 Answer (B) is correct Payments should be made within discount periods if the return is more than the firm's cost of capital With terms of 3/10, net 45, the buyer is earning a 3% savings for paying on the tenth day, or 35 days earlier than would otherwise be required For example, on a $1,000 invoice, the payment would be only $970 The $30 savings is comparable to interest earned on a $970 loan to the vendor (the payment is not due for another 35 days) The interest rate on this hypothetical loan is 3.09278% ($30/$970) That return is for a 35-day period Annualizing the return requires determining the number of 35-day periods in a year Multiplying the return for 35 days times the periods in a year results in an annual rate of return of about 31.81% [3.09278% x (360 days/35 days)] Answer (A) is incorrect because 55.67% is based on terms of 3/10, net 30 Answer (C) is incorrect because 22.27% is based on an earning period of 50 days Answer (D) is incorrect because the discount is 3%, not 9% 108 Answer (B) is correct The company will initially lose $2 by not taking the discount This amount is partially offset by interest earned on $98 for 50 days of $.817 Thus, the net cost is $1.183 ($2.00 - $.817) Since a 360-day year has 7.2 fifty-day periods, the total annualized cost is $8.52 (7.2 x $1.183) The loss rate is about 8.7% ($8.52/$98) Answer (A) is incorrect because the cost of not taking the cash discount is 8.7% The company will initially lose $2 by not taking the discount This amount is partially offset by interest earned on $98 for 50 days of $.817 Thus, the net cost is $1.183 ($2.00 - $.817) Because a 360-day year has 7.2 periods of 50 days each, the total annualized cost is $8.52 (7.2 x $1.183) The loss rate is about 8.7% ($8.52/$98) Answer (C) is incorrect because the cost of not taking the cash discount is 8.7% The company will initially lose $2 by not taking the discount This amount is partially offset by interest earned on $98 for 50 days of $.817 Thus, the net cost is $1.183 ($2.00 - $.817) Because a 360-day year has 7.2 periods of 50 days each, the total annualized cost is $8.52 (7.2 x $1.183) The loss rate is about 8.7% ($8.52/$98) Answer (D) is incorrect because the cost of not taking the cash discount is 8.7% The company will initially lose $2 by not taking the discount This amount is partially offset by interest earned on $98 for 50 days of $.817 Thus, the net cost is $1.183 ($2.00 - $.817) Because a 360-day year has 7.2 periods of 50 days each, the total annualized cost is $8.52 (7.2 x $1.183) The loss rate is about 8.7% ($8.52/$98) 109 Answer (C) is correct Assume a $1,000 invoice The discount would be $50, and the customer would have to pay $950 Thus, the customer is paying $50 for the extended use of $950 a rate of 5.26% for 50 days Extrapolating this to a full year involves multiplying by the number of 50-day periods in a year, which is 7.3 (365 ÷ 50) Thus, 5.26% x 7.3 periods gives an annual rate of 38.4% Answer (A) is incorrect because 30.64% is the result of assuming the use of $1,000 for 60 days Answer (B) is incorrect because 36.60% is the result of assuming the use of $1,000 instead of $950 Answer (D) is incorrect because 45.39% is the result of using the wrong number of days 110 Nominal percentage cost = 111 Nominal percentage cost = 360 x = 36.7% 98 35 - 15 360 x = 21.41% 97 52 112 Calculate the nominal percentage, which is the nominal annual cost: Nominal cost = 360 days × = 0.0204  36 = 0.7344 = 73.44% 100 − 20 − 10 Calculate the effective annual rate (EAR): Numerical solution: EAR = (1.0204)36 - 1.0 = 2.0689 - 1.0 = 106.89%  106.9% Financial calculator solution: (EAR) Inputs: P/YR = 36; NOM% = 73.44 Output: EFF% = 106.89%  106.9% 113 The company pays every 45 days or 360/45 = times per year Thus, the average accounts payable are $5,000,000/8 = $625,000 The effective cost of trade credit can be found as follows: EAR = (1 + 2/98)360/30 - = 1.2743 - = 0.2743 = 27.43% 114 REQUIRED: The weighted-average annual interest rate.DISCUSSION: If the company pays Web Master within 10 days, it will save $500 (2% x $25,000) Thus, the company is effectively paying $500 to retain $24,500 ($25,000 - $500) for 20 days (30 – 10) The annualized interest rate on this borrowing is 36.7346% [($500 ÷ $24,500) x (360 days ÷ 20 days)] Similarly, the company is, in effect, paying Softidee $2,500 (5% x $50,000) to hold $47,500 ($50,000 - $2,500) for 80 days (90 – 10) The annualized rate on this borrowing is 23.6842% [($2,500 ÷ $47,500) x (360 days ÷ 80 days)] The weighted-average of these two rates based on average monthly purchases is 28.0% {[36.7346% x ($25,000 ÷ $75,000)] + [23.6842% x ($50,000 ÷ $75,000)]} This calculation, however, understates the true cost of not taking the discount because it does not consider the effects of compounding Answer (A) is incorrect because 27.0% is based on gross purchases Answer (C) is incorrect because 29.3% is an unweighted (simple) average based on gross purchases Answer (D) is incorrect because 30.2% is an unweighted average of the two interest rates 115 REQUIRED: The true statement about the decision to use trade credit and pay at the end of the credit period DISCUSSION: (D) The company is currently paying an annual rate of 28% (see previous question) to obtain trade credit and pay at the end of the credit period This policy should be continued if trade credit is the only source of financing, or if other sources are available only at a higher rate Answer (A) is incorrect because the company should continue the current practice unless alternative short-term financing is available at a lower rate Answer (B) is incorrect because the weighted-average cost of capital is usually a concern in capital budgeting and is not as important in the decision process as the marginal cost of capital Answer (C) is incorrect because the company should maintain its current practice if the cost of alternative long-term financing is higher 116 Funds required 1.0 - Nominal rate (decimal) $100,000 $100,000 = = = $112,359.55  $112,360 1.0 - 0.11 0.89 Face value = 117 Convert the annual rate to a periodic rate (quarterly) in the denominator of the face value formula: Funds required Face value = 1.0 - Nominal rate _ 90 / 360 = $60,000 $60,000 = = $61,855.67  $61,856 1.0 - 0.12(0.25) 0.97 118 Answer (B) is correct Simple interest is charged on the amount actually paid to the borrower If interest is charged on a simple basis, the full $20,000 face value of the loan is made available to the borrower Answer (A) is incorrect because interest is charged on a discount basis when it is deducted from the face value borrowed Answer (C) is incorrect because interest charged on a discount basis results in a deduction from the face value The borrower does not receive the full face value of the loan Answer (D) is incorrect because interest is charged on an add-on basis when the face value of the loan initially equals the borrowed amount plus the nominal interest charge 119 Total to be repaid = $12,000(1.1019) = $13,222.80 Interest = $13,222.80 - $12,000 = $1,222.80 Approximate rateAdd-on = 120 The monthly payments would be: Monthly payment = 121 $1,222.80 = 0.2038 = 20.38% $12,000 / $75,000 + $7,500 = $6,875 12 $7,500 = 20% $75,000 / Approximate rate = 122 Answer (B) is correct At 10%, the interest on a $500,000 loan is $50,000 per year However, the $500,000 loan is effectively reduced to $450,000 of usable funds by the compensating balance requirement Thus, the borrower pays $50,000 of interest for a $450,000 loan, an effective rate of 11.1% ($50,000/$450,000) Answer (A) is incorrect because 10% is the nominal rate Answer (C) is incorrect because 9.1% equals $50,000 divided by $550,000 Answer (D) is incorrect because 12.2% equals $55,000 divided by $450,000 123 Answer (B) is correct The compensating balance is $750,000, which comes from the product of $5,000,000 and 0.15 The interest charged is $5,000,000 x 0.1 = $500,000 Thus, the amount of funds available for use is $4,250,000 The cost of the loan is the interest cost of $500,000 divided by the effective amount of money available for use: Effective Cost = $500,000 ÷ $4,250,000 = 0.1176 = 11.76% Answer (A) is incorrect because the effective cost of the loan is not equal to the annual rate of interest Answer (C) is incorrect because 16.67% is the result of dividing the compensating balance by $4,500,000 (amount of interest charged minus the interest) Answer (D) is incorrect because 17.65% is the result of dividing the compensating balance by $4,250,000 (which is the interest minus the compensating balance) 124 Answer (D) is correct Annual interest is $36,000 ($300,000 x 12%), and the amount available is $255,000 [$300,000 - (15% x $300,000)] Thus, the effective interest rate is 14.12% ($36,000 ÷ $255,000) Answer (A) is incorrect because 11.00% is the nominal rate for 11 months Answer (B) is incorrect because 12.00% is the nominal rate of interest Answer (C) is incorrect because 12.94% equals $33,000 (11 months of interest) divided by $255,000 125 REQUIRED: The effective cost of a loan when a compensating balance is required DISCUSSION: Interest on the loan is $7,000 (7% x $100,000) Given that the borrower has to maintain a 20% compensating balance, only $80,000 [$100,000 – (20% x $100,000)] is available for use Thus, the company is paying $7,000 for the use of $80,000 in funds at an effective cost of 8.75% ($7,000 ÷ $80,000) Answer (A) is incorrect because the borrower has access to less, not more, than the face amount of the loan Answer (B) is incorrect because the effective rate is higher than the contract rate as a result of the compensating balance requirement Answer (C) is incorrect because 8.40% is 120% of the contract rate 126 REQUIRED: The effective interest rate given a compensating balance requirement DISCUSSION: (D) The requirement to maintain a compensating balance of 20% of the $300,000 loan means that the borrower has effective use of only 80% of the loan or $240,000 The 8% interest rate applied to a $300,000 lon requires an annual interest expenditure of $24,000 In turn, paying $24,000 for the use of $240,000 indicates an effective interest rate of 10% Answer (A) is incorrect because having only 80% of the borrowed funds available means the effective rate will be greater than the 8% contract rate Answer (B) is incorrect because 8% is the nominal rate Answer (C) is incorrect because 20% is the percentage of the required compensating balance 127 REQUIRED: The effective interest rate on a loan that requires a compensating balance of $25,000 above the company’s normal working balance DISCUSSION: The $50,000 compensating balance requirement is partially satisfied by the company’s practice of maintaining a $25,000 balance for transaction purposes Thus, only $25,000 of the loan will not be available for current use, leaving $225,000 of the loan usable At 6% interest, the $250,000 loan would require an interest payment of $15,000 per year This is partially offset by the 2% interest earned on the $25,000 incremental balance, or $500 Subtracting the $500 interest earned from the $15,000 of expense results in net interest expense of $14,500 for the use of $225,000 in funds Dividing $14,500 by $225,000 produces an effective interest rate of 6.44% Answer (B) is incorrect because 7.00% fails to consider that the $25,000 currently being maintained counts toward the compensating balance requirement Answer (C) is incorrect because 5.8% fails to consider the compensating balance requirement Answer (D) is incorrect because 6.66% fails to consider the interest earned on the incremental balance being carried 128 REQUIRED: The effective interest rate on a loan given a compensating balance requirement DISCUSSION: (D) Of the $500,000 borrowed, the debtor has the use of only $450,000 The compensating balance provision requires a minimum balnace that is $50,000 greater than the balance the company usually maintains At 8% on $500,000 loan, the annual interest expense is $40,000 However, this amount is reduced by the interest earned on the extra $50,000 in the checking account At 3%, the extra $50,000 earns $1,500 per year Thus, the net expense is $38,500 The effective interest rate is 8.555% ($38,500 ÷ $450,000) Answer (A) is incorrect because the effective interest rate must exceed the 8% contract rate because not all of the borrowed funds are available for the debtor’s use Answer (B) is incorrect because 8.22% assumes incremental earnings on the checking account of $3,000 Answer (C) is incorrect because 9.25% is based on the assumption that the company ordinarily maintains a zero balance 129 Answer (D) is correct Applying the 9% interest rate to a $100,000 loan results in interest expense of $9,000 If the loan is processed in the form of a discounted note, the interest will be deducted from the proceeds of the loan Thus, the $9,000 of interest will be deducted from the $100,000 note, resulting in loan proceeds of $91,000 The borrower is paying $9,000 for a loan of $91,000, resulting in an effective interest rate of 9.89% Answer (A) is incorrect because the lesser amount of funds available on a discounted note means the effective rate will be higher than the contract rate Answer (B) is incorrect because 9% is the nominal rate (discount rate) Answer (C) is incorrect because 9.81% equals the nominal rate multiplied by 9% 130 Answer (D) is correct If the loan is discounted, the borrower receives the face amount minus the prepaid interest Thus, the borrower will receive proceeds of $1,800,000 [$2,000,000 - (10% x $2,000,000)] The effective interest rate is 11.11% ($200,000 ÷ $1,800,000) Answer (A) is incorrect because the prepayment of interest reduces the funds available, resulting in an effective interest rate greater than the contract rate Answer (B) is incorrect because the prepayment of interest reduces the funds available, resulting in an effective interest rate greater than the contract rate Answer (C) is incorrect because 10% is the contract rate The effective rate is higher because the full $2 million face amount of the note will not be available to the borrower 131 Answer (D) is correct When a note is discounted, the interest is deducted from the face of the note and is never received by the borrower At a 10% interest rate, the interest will be $100,000, meaning the borrower will receive only $900,000 A cost of $100,000 on $900,000 of available funds results in an effective rate of 11.1% Answer (A) is incorrect because only $900,000 is available for use Answer (B) is incorrect because only $900,000 is available for use Answer (C) is incorrect because the effective rate is 11.1% 132 Answer (C) is correct Under a discounted loan, the interest is subtracted from the face of the note to determine the proceeds Thus, on a $30,000 note at 11%, the proceeds would be only $26,700 Dividing the $3,300 of interest by the $26,700 proceeds results in an effective rate of 12.36% Answer (A) is incorrect because the effective rate on a discounted note will be higher than the contract rate Answer (B) is incorrect because 10% results from adding the interest to the face of the note instead of deducting it Answer (D) is incorrect because 11% is the contract rate, not the effective rate 133 Answer (C) is correct Discount interest is subtracted before the loan proceeds are paid to the borrower A compensating balance is an amount that the borrower must keep on deposit with the lender The effective annual interest rate is increased by both the discount interest arrangement and by the compensating balance requirement The effective rate equals the nominal rate dividend by one minus the sum of the nominal rate and the compensating balance percentage, or 20% [.15 ÷ (1.0 - 15 - 10)] Answer (A) is incorrect because 16.67% does not adjust for the discount interest arrangement Answer (B) is incorrect because 17.65% does not adjust for the compensating balance requirement Answer (D) is incorrect because 25.00% is the sum of the nominal rate and the compensating balance percentage 134 Answer (D) is correct Assuming a $1,000 loan, the interest at 7% for year is $70 Hence, the proceeds of the loan are $930 ($1,000 - $70) Also, 20% of the note, or $200, cannot be used by the borrower because of the compensating balance requirement Consequently, only $730 is available for use by the borrower Paying $70 interest for the use of $730 gives an interest rate of $9.59% ($70/$730) Answer (A) is incorrect because 8.75% equals $70 divided by $800 Answer (B) is incorrect because 9.41% equals $70 divided by $744 Answer (C) is incorrect because 7.53% equals $70 divided by $930 135 Answer (D) is correct Assuming a $1,000 loan, the interest at 6% for year is $60 Hence, the proceeds of the loan with discounted (paid-in-advance) interest are $940 ($1,000 - $60) Also, 22% of the note, or $220, cannot be used by the borrower because of the compensating balance requirement Consequently, only $720 is available for use Paying $60 interest for the use of $720 results in an effective cost of borrowing of 8.33% ($60 ÷ $720) Answer (A) is incorrect because 6% is the contract rate Answer (B) is incorrect because 6.38% does not consider the compensating balance requirement Answer (C) is incorrect because 7.69% assumes the interest is not paid in advance 136 Answer (B) is correct An amount-of-one table can be used to find the effective rate when the APR is known An annual rate of 9% is equivalent to 75% per month Using the 75% column, the accumulated interest factor is 093806, or 9.38% for the year Answer (A) is incorrect because 9% is the APR Answer (C) is incorrect because 9.81% is too high Answer (D) is incorrect because 10.94% is a nonsense answer 137 Answer (A) is correct The annual percentage rate is easy to compute: multiply the 1.25% monthly rate times 12 months to arrive at 15% The compounded rate would be slightly higher Thus, given the alternatives, answer (A) is the only acceptable alternative where the compounded rate is greater than the 15% APR To check your solution, look at an "Amount of 1" table for 12 periods at 1.25% The accumulated interest is 16075, or 16.8% Answer (B) is incorrect because it has both amounts wrong Answer (C) is incorrect because the compounded rate will be greater than the 15% APR Answer (D) is incorrect because it substitutes the APR for the compounded rate 138 REQUIRED: The effective interest rate on the borrowing DISCUSSION: (B) The effective interest rate on the 180-day borrowing is equal to the net interest cost divided by the net available proceeds of $1,800,000 ($2,000,000 loan – $200,000 increase in the compensation balance) The net interest cost is equal to the gross interest cost minus the incremental interest revenue The gross interest cost is $120,000 [$2,000,000 x 12% x (6 ÷ 12)] Because the incremental interest revenue is $6,000 [$200,000 x 6% x (6 ÷ 12)], the net interest cost if $114,000 ($120,000 – $6,000) The six-month effective interest rate is therefore 6.33% ($114,000 ÷ $1,800,000) The annual effective interest rate is 12.67% (6.33% x 2) Answer (A) is incorrect because 12% is the annual rate Answers (C) and (D) are incorrect because the interest revenue from the checking account must be included in the calculations 139 Answer (B) is correct The total cost to the company will be $21,200 ($20,000 discount + $1,200 of transaction costs), and the net amount available will be $978,800 The annualized amount of the costs is $84,800 (4 x $21,200) Accordingly, the annual interest cost will be 8.66% ($84,800/$978,800) Answer (A) is incorrect because the annual interest cost will be 8.66% Answer (C) is incorrect because the annual interest cost will be 8.66% Answer (D) is incorrect because the annual interest cost will be 8.66% 140 Answer (C) is correct The total cost to the company will be $51,500 (50,000 discount + 1,500 of transaction costs), and the net amount available will be $1,448,500 The annualized amount of the costs is $206,000 (4 x $51,500) Accordingly, the annual interest cost will be 14.22% ($206,000 ÷ $1,448,500) Answer (A) is incorrect because 3.45% ignores the transaction costs and fails to annualize the percentage Answer (B) is incorrect because 3.56% fails to annualize the result Answer (D) is incorrect because 13.79% ignores the transaction costs 141 142 143 Interest rate on borrowed funds = 0.09 + 0.015 = 10.5% Cost of unused portion: $4,000,000  0.005 = $ 20,000 Cost of used portion: $6,000,000  0.105 = 630,000 Total cost of loan agreement $650,000 Answer (C) is correct At 10% interest, the annual expense would be $120,000 on a loan of $1.2 million However, because of the 15% compensating-balance requirement, $180,000 of the funds cannot be used Therefore, usable funds are only $1,020,000 Dividing the $120,000 of interest by the $1,020,000 results in an effective rate of 11.76% Answer (A) is incorrect because 10% is the contract rate, not the effective rate Answer (B) is incorrect because 11.11% results from assuming a 10% compensating balance Answer (D) is incorrect because 13.33% results from assuming a $300,000 compensating balance instead of $180,000 REQUIRED: The interest expense for six months DISCUSSION: The sum of the beginning balance and inflows exceeds the outflows for the first months At the end of March, however, Garth must use $2,000,000 of its line of credit ($2,000,000 beginning balance + $6,000,000 inflows - $100,000,000 outflows) Thus, interest for April is $20,000 (1% x $2,000,000) The net cash outflow for April (ignoring short-term borrowings) is $1,000,000 of an additional $1,000,000 of the line of credit) However, the $20,000 of interest for April must also be paid, so the amount of the line of credit used in May is $3,020,000 ($2,000,000 + $1,000,000 + $20,000) Interest for May is therefore $30,200 (1% x $3,020,000) Given the net cash inflow for May of $2,000,000 (again ignoring short-term borrowings) and the borrowing of $30,200 to pay the interest for May, the amount of the line of credit used in June is $1,050,200 Interest in June is $10,502 (1% x $1,050,200), and total interest is $60,702 ($20,000 + $30,200 + $10,502) Consequently, the closest answer is $61,000 Answer (A) is incorrect because interest must be paid monthly when the credit line is used in April, May, and June Answers (C) and (D) are incorrect because the company would repay the credit line at the end of months with a positive cash flow 144 Answer (D) is correct The first step is to determine the actual annual percentage interest rate for each of the four options Assuming a $100 invoice, the Fort Company discount represents interest of $1 on a loan of $99 for 20 days (30-day credit period - 10-day discount period) The annual interest rate is 18.1818% [(360/20) periods x ($1/$99)] The Riley Company discount represents an interest charge of $2 on a loan of $98; i.e., by not paying on the 15th day, the company will have the use of $98 for 45 days (60-day credit period - 15-day discount period) The number of periods in a year would be (360/45) The interest would be 16.326% ($2/$98 x periods) The Shad loan would be for $97 at a cost of $3 The loan would be for 75 days (90 - 15) Given 4.8 interest periods in a year (360/75), the annual interest rate would be 14.845% ($3/$97 x 4.8) The bank loan was quoted at 14.75% on a discount basis On a $100 note, the borrower would only receive $85.25, giving an interest rate of 17.302% ($14.75/$85.25) Thus, not paying Shad, Inc.'s invoices on time would be the lowest cost source of capital, at a cost of 14.845% Answer (A) is incorrect because the actual annual percentage rate based on forgoing Shad's discount is 14.845% This is lower than the rate on the bank loan (17.302% because it is a discount loan), or the cost of forgoing the discounts allowed by Fort Co (18.182%) and Riley Co (16.326%) Answer (B) is incorrect because the actual annual percentage rate based on forgoing Shad's discount is 14.845% This is lower than the rate on the bank loan (17.302% because it is a discount loan), or the cost of forgoing the discounts allowed by Fort Co (18.182%) and Riley Co (16.326%) Answer (C) is incorrect because the actual annual percentage rate based on forgoing Shad's discount is 14.845% This is lower than the rate on the bank loan (17.302% because it is a discount loan), or the cost of forgoing the discounts allowed by Fort Co (18.182%) and Riley Co (16.326%) 145 Answer (B) is correct The interest cost of borrowing $4,900,000 (98% x $5,000,000) to take advantage of the discount is $24,500 [$4,900,000 x 12% x (15 ÷ 360)], and the total cost will be $4,924,500 The total cost if the discount is not taken will be $5,000,000, a difference of $75,500 Answer (A) is incorrect because $51,000 less is based on a 30-day borrowing period Answer (C) is incorrect because $100,000 less does not consider the interest paid Answer (D) is incorrect because $24,500 more reflects interest paid but ignores the discounted price 146 Answer (B) is correct The corporation can obtain trade credit for 20 additional days by not paying within the discount period Instead of paying $99,000 to satisfy its obligation within 10 days, it can pay $100,000 at the end of 30 days The corporation will thus incur $1,000 in interest to hold the $99,000 for the 20 days Because a 360-day year has 18 such periods, the interest rate is approximately 18.18% [($1,000 ÷ $99,000) x 18] However, if compounding effects are considered, the rate is higher The effective rate, taking compounding into consideration, is found using the following formula: 18   Effective rate = 1 +  − = 19.83%  99  Answer (A) is incorrect because the trade credit rate is 18.18% or 19.83% depending on the method of calculation Answers (C) and (D) are incorrect because the note has an effective rate, including compounding effects, of 21.94% The following is the calculation: 12 0.20   Effective rate = 1 +  12   - = 21.94% 147 Answer (A) is correct The company will save $200 (2% x $10,000) every 15 days There are 24 15-day periods in a 360-day year Thus, the gross savings will be $4,800 The interest expense on a $9,800 loan at 12% is $1,176 Deducting the $1,176 interest expense from the $4,800 of discounts results in a net savings of $3,624 Answer (B) is incorrect because $1,176 is the amount of the interest expense Answer (C) is incorrect because $4,800 is the gross savings before deduction of interest Answer (D) is incorrect because $1,224 is based on only $2,400 of discounts, instead of $4,800 148 Calculate A/P with and without taking discounts: A/PNo discount = $11,760 x 30 days = $352,800 A/PDiscount = $11,760 x 10 days = $117,600 Calculate financing amount in notes payable and interest cost The firm will need to borrow the difference in notes payable $352,800 - $117,600 = $235,200 The additional interest cost is $235,200 x 0.10 = $23,520 Calculate total purchases and discounts lost: Total purchases = 360 days x 12,000 gross purchases = $4,320,000 Discounts lost = $4,320,000 x 0.02 = $86,400 Construct comparative financial statements: I Partial balance sheet:Take DiscountsDon’t Take Discounts (Borrow N/P) (Use Max Trade Cdt) DifferenceAccounts payable$117,600 $352,800 -$235,200Notes payable (10%) 235,200 +235,200 Total current liab.$352,800 $352,800 $ II Partial income statementEBIT & discounts lost $140,000$140,000 $ 0Less: Interest -23,5200 +23,520 Discounts lost -86,400 -86,400EBT116,480 53,600 +62,880Less: Taxes (at 40%) 46,592 21,440 +25,152Net income $ 69,888 $ 32,160 $ 37,728 149 Answer (D) is correct The company will need $49,000 (98% x $50,000) to pay off the invoice In addition, it will need a compensating balance equal to 10% of the loan This can be written in equation form as: Loan = $49,000 + (.1)Loan Thus, the loan amount needed is $54,444 ($49,000/.9) Answer (A) is incorrect because $55,000 is 110% of the invoice Answer (B) is incorrect because $55,056 is a nonsense number Answer (C) is incorrect because $55,556 assumes no cash discount 150 Answer (B) is correct The company will need $49,000 (98% x $50,000) to pay off the invoice In addition, it will need a compensating balance equal to 10% of the loan Thus, the loan amount needed is $54,444 ($49,000/.9).The interest at 12% annually on a 30-day loan of $54,444 is $544.44 ($54,444 x 12% x 30/360) However, the company has access to only $49,000 Thus, the interest expense on usable funds is at an annual rate of 13.33% (12 months x $544.44/$49,000) Answer (A) is incorrect because 12.00% assumes that the company has access to loan funds of $54,444 Answer (C) is incorrect because $54,444, not $55,000, is the amount of the loan, and the annual interest expense should be divided by $49,000 of usable funds, not $50,000 Answer (D) is incorrect because $54,444, not $55,056, is the amount of the loan used to determine annual interest expense 151 Answer (D) is correct If the company chooses the line of credit, it will pay 17% interest on $80,000 ($100,000 $20,000 discount) and 1% on the $20,000 unused portion, a total of $13,800 The effective interest rate would thus be 17.25% ($13,800 ÷ $80,000) Answer (A) is incorrect because if the company forgoes the cash discount, its effective rate is 25% ($20,000 ÷ $80,000 immediate cash price) Answer (B) is incorrect because an 18% effective rate exceeds that on the line of credit Answer (C) is incorrect because the effective rate would be 18.75% [(15% x $100,000) ÷ $80,000 available funds] 152 Answer (C) is correct The company will need $58,800 (98% x $60,000) to pay the invoice In addition, it will need a compensating balance equal to 9% of the loan The equation is Loan = $58,800 + 09 Loan Thus, the loan amount needed is $64,615 ($58,800 ÷ 91) Answer (A) is incorrect because $60,000 is the invoice amount Answer (B) is incorrect because $65,934 assumes the amount paid to the supplier is $60,000 Answer (D) is incorrect because $58,800 is the amount to be paid to the supplier 153 Answer (C) is correct The interest at 11% annually on a 30-day loan of $64,615 is $592.30 [$64,615 x 11% x (30 ÷ 360)] However, the company has access to only $58,800 The interest expense on usable funds is therefore at an annual rate of 12.09% [12 months x ($592.30 ÷ $58,800)] Answer (A) is incorrect because 11% is the contract rate of interest Answer (B) is incorrect because the effective rate is greater than the contract rate The usable funds are less than the face amount of the note Answer (D) is incorrect because the effective rate is greater than the contract rate The usable funds are less than the face amount of the note 154 Answer (C) is correct By failing to take the discount, the company is essentially borrowing $58,800 for 30 days Thus, at a cost of $1,200, the company acquires the use of $58,800, resulting in a rate of 2.04081% ($1,200 ÷ $58,800) for 30 days Assuming a 360-day year, the effective annual rate is 24.489% [2.04081% x (360 days ÷ 30 days)] Answer (A) is incorrect because 2% is the discount rate for a 30-day period Answer (B) is incorrect because 24% assumes that the available funds equal $60,000 Answer (D) is incorrect because 36.73% assumes a 20-day discount period 155 Answer (D) is correct If the loan is processed in the form of a discounted note, the interest will be deducted from the proceeds of the loan Thus, on the 10% loan, the $10,000 of interest will be deducted from the $100,000 note, resulting in loan proceeds of $90,000 The borrower is paying $10,000 for a loan of $90,000, resulting in an effective interest rate of 11.11% On the 10.5% loan, the $10,500 of interest will be deducted from the $100,000 note, resulting in proceeds of $89,500 The borrower is paying $10,500 for a loan of $89,500, resulting in an effective interest rate of 11.73% Answer (A) is incorrect because Bank has an effective interest rate of 10.49% Answer (B) is incorrect because Bank also has an effective interest rate between 11% and 12% Answer (C) is incorrect because Bank has an effective interest rate of 9.89% 156 Answer (A) is correct The effective cost of a loan is found by dividing the interest on the loan by the amount available for use Therefore, the interest charged by Bank is $7,000 (7% x $100,000) Given that the borrower has to maintain a 20% compensating balance, only $80,000 [$100,000 - (20% x $100,000)] is available for use Thus, the company is paying $7,000 for the use of $80,000 in funds at an effective cost of 8.75% ($7,000 ÷ $80,000) Answer (B) is incorrect because Bank offers an effective loan cost of 9.41% Answer (C) is incorrect because Bank offers an effective loan cost of 9.15% Answer (D) is incorrect because Bank offers an effective loan cost of 9.00% 157 Answer (A) is correct Since Bank has a 25% compensating balance requirement, the amount of available funds is $3,750,000 (75% x $5 million) Interest at 12% is $600,000 Dividing $600,000 by $3,750,000 results in an effective rate of 16.0% For Bank 2, the interest is $750,000 Dividing $750,000 in interest by $4.5 million of available funds results in an effective rate of 16.7% Thus, the rate on the loan from Bank is 0.7% less than the rate on the loan from Bank Answer (B) is incorrect because 3% is the difference in the two contract rates, not the effective rates Answer (C) is incorrect because the effective rate for Bank is lower than for Bank Answer (D) is incorrect because the effective rate for Bank is lower than for Bank 158 Answer (A) is correct If the firm plans to have $500,000 on deposit in any case, the Bank loan will require only $750,000 more to maintain the compensating balance, meaning that $4,250,000 of the loan will be available for use Dividing $600,000 by $4,250,000 results in an effective rate of 14.1% All of the loan from Bank would be available, resulting in an effective rate equal to the 15% contract rate Answer (B) is incorrect because 3% is the difference in contract rates, not the effective rates Answer (C) is incorrect because Bank has a lower effective rate Answer (D) is incorrect because Bank has a lower effective rate 159 Answer (A) is correct To make a capital budget decision, the present values of the inflows must be determined and compared to see which is the greater value The lump sum payment is a one-time payment at a set date at a specified interest rate The formula to determine the present value of the lump sum is $20,000 x (1 + i) n Where i = interest rate, 005, and n = number of periods, 26 bi-weekly periods The computation of the present value of the lump sum is as follows: (1 + 0.005) 26 $20, 000 x 1.13846 $20, 000 x 0.87838 = $17,567.60 ~ $17,568 $20,000 x The present value of the bi-weekly payments can be calculated using the formula for the present value of an ordinary annuity, as the interest compounds bi-weekly The formula is: $750 x - [1 ÷ (1 + i) n i Where i = interest rate, 005, and n = number of periods, 26 bi-weekly periods The computation of the present value of the bi-weekly payments is as follows: - [1 ÷ (1 + 0.005) 26 $750 x 0.005 - [1 ÷ (1.13846)] $750 x 0.005 - 0.87838 $750 x 0.005 $750 x 24.324 = $18,243 By comparing the present values of the options ($17,568; $18,243), Roger should choose the second option of biweekly payments, as the present value is higher by $675 Answer (B) is incorrect because the two options will have different present values Answer (C) is incorrect because the biweekly payments have a higher present value Answer (D) is incorrect because the biweekly payments have a higher present value 160 REQUIRED: The annual percentage cost of factoring receivables DISCUSSION: (D) The factor will advance $100,000 (80% of $125,000) This amount is the average balance outstanding throughout the year Thus, annual interest will be $10,000 (10% x $100,000) In addition, the company will pay an annual fee of $30,000 (2%x $125,000 per month x 12 months), so the total annual net cost is $16,000 ($10,000 + $30,000 - $24,000 savings) Hence, the annual cost is 16% ($16,000 ÷ $100,000) Answer (A) is incorrect because 10% is the interest rate on the amount advanced Answer (B) is incorrect because 12% is the sum of the interest rate and the fee percentage Answer (C) is incorrect because 13.2% is the cost of Option B 161 REQUIRED: The annual percentage cost of borrowing with a compensating balance requirement DISCUSSION: (C) Even though the company will borrow $110,000, it will have use of only $100,100 because a 9% compensating balance, or $9,900, must be maintained at all times Consequently, the effective annual interest rate is 13.2% [(12% x $110,000) ÷ $100,000] Answer (A) is incorrect because 9% is the compensating balance requirement Answer (B) is incorrect because 12% is the contract rate Answer (D) is incorrect because 21% is the sum of the contract rate and the compensating balance requirement 162 REQUIRED: The annual percentage cost of issuing commercial paper DISCUSSION: (D) By issuing commercial paper, the company will receive $100,000 and repay $110,000 every six months Thus, for the use of $100,000 in funds, the company pays $10,000 in interest each six-month period or a total of $20,000 per year The annual percentage rate is therefore 20% ($20,000 ÷ $100,000) Answer (A) is incorrect because 9.1% is the 6-month rate based on the face amount of the paper Answer (B) is incorrect because 10% is the rate for six months Answer (C) is incorrect because 18.2% is based on the face amount of the commercial paper 163 REQUIRED: The annual percentage cost of a discounted note DISCUSSION: (B) The company will receive $100,000 (80% x $125,000) at an annual cost of $25,000), so the annual interest rate is 25% ($25,000 ÷ $100,000) Answer (A) is incorrect because the effective rate must exceed the contract rate of 20% Answer (C) is incorrect because 40% assumes no discount and a 6-month loan term Answer (D) is incorrect because 50% assumes a 6month loan term ... unaffected by the change in policy.) (E) A Decrease by $4,000 C Decrease by $240,000 B Decrease by $40,000 D Decrease by $280,000 CIA 1196 IV-44 Page 15 of 58 MANAGEMENT ADVISORY SERVICES WORKING CAPITAL. .. compensating balance 157 Which bank offers the better deal, expressed on an annual basis? (M) A Bank is better by 7% C Bank is better by 7% B Bank is better by 3.0% D Bank is better by 2.0% 158 Which would... QUESTIONS Bank has a stated interest rate of 9.5%, Bank has a stated interest rate of 10%, and Bank has a stated interest rate of 10.5%? (M) A Bank only C Either Bank or Bank B Bank only D Either Bank

Ngày đăng: 28/02/2018, 15:43

Xem thêm:

TỪ KHÓA LIÊN QUAN

Mục lục

    Effect of Plant Expansion on Working Capital

    WORKING CAPITAL FINANCING POLICY

    Working Capital Policy Options

    Other Cash Management Systems

    Economic Conversion Quantity (ECQ)

    Annually Compounded Rate of Return

    Yield on Floating-rate Preferred Stock

    Carrying Cost on Accounts Receivable

    Days Receivable & Average Accounts Receivable Balance

    Aging of Accounts Receivable

TÀI LIỆU CÙNG NGƯỜI DÙNG

TÀI LIỆU LIÊN QUAN

w